1. 基础知识
1.1. 术语
因数(Factors):因数是指相乘得到乘积的数。
\( 1 \times {12} = 2 \times 6 = 3 \times 4 = {12} \Rightarrow 1,2,3,4,6 \) 和 12 都是 12 的因数。
除数(Divisor):除数是指能整除给定数且不留余数的数。
\( \frac{6}{1} = 6\;\frac{6}{2} = 3\;\frac{6}{3} = 2\;\frac{6}{6} = 1. \)
分母 1、2、3 和 6 被称为分子 6 的除数。
注意:一个数的因数与该数的除数含义相同。
倍数(Multiple):倍数是指将两个或更多数相乘得到的乘积。
\( {24} = 1 \times {24} = 2 \times {12} = 3 \times 8 = 4 \times 6 \Rightarrow {24} \) 是 1、2、3、4、6、8、12 和 24 的倍数。
质因数分解(Prime factorizations):质因数分解是将一个数表示为所有因数均为质数的乘积。
\( {2016} = {32} \times {63} = {2}^{5} \times {3}^{2} \times 7. \) \( {1001} = 7 \times {11} \times {13}. \)
1.2. 定理
定理 1. 算术基本定理(Fundamental theorem of arithmetic)
任何合数(除 0 和 1 外)都可以唯一地表示为质数的乘积。
定理 2:
若 \( a\left| {b, a}\right| c \) ,则 \( a \mid \left( {{jb} + {kc}}\right) \) 和 \( j, k \) 为任意整数。
1.3. 除数公式
约数个数
对于大于1的整数 \( n \) ,设其质因数分解为
\( n = {p}_{1}^{a}{p}_{2}^{b}{p}_{3}^{c}\ldots {p}_{k}^{m} \) ,其中 \( a, b, c,\ldots \) 和 \( m \) 为非负整数, \( {p}_{1},{p}_{2},\ldots ,{p}_{\mathrm{k}} \) 为质数,则约数个数为:
\( d\left( n\right) = \left( {a + 1}\right) \left( {b + 1}\right) \left( {c + 1}\right) \ldots \left( {m + 1}\right) \) (24.1)
偶约数个数与奇约数个数
对于 \( n = {2}^{j}{p}_{1}^{a}{p}_{2}^{b}{p}_{3}^{c}\ldots {p}_{k}^{m} \) ,其中 \( j, a, b, c,\ldots \) 和 \( m \) 为非负整数,且
\( {p}_{1},{p}_{2},\ldots ,{p}_{\mathrm{k}} \) 为除2以外的质数,
若 \( j = 0 \) ,则偶约数个数为零。
若 \( j = 1 \) ,则偶约数个数为: \( {d}_{e}\left( n\right) = \left( {a + 1}\right) \left( {b + 1}\right) \left( {c + 1}\right) \ldots \left( {m + 1}\right) \)
若 \( j > 1 \) ,则偶约数个数为: \( {d}_{e}\left( n\right) = j\left( {a + 1}\right) \left( {b + 1}\right) \left( {c + 1}\right) \ldots \left( {m + 1}\right) \)
奇约数个数为 \( {d}_{o}\left( n\right) = \left( {a + 1}\right) \left( {b + 1}\right) \left( {c + 1}\right) \ldots \left( {m + 1}\right) \)
其中 \( d\left( n\right) = {d}_{e}\left( n\right) + {d}_{o}\left( n\right) \)
为 \( m \) 的倍数的约数个数
步骤:(1) 对给定数进行质因数分解;(2) 取出一个 \( m \) ;(3) 计算剩余数的因数个数。
为平方数的约数个数
步骤:(1) 对给定数进行质因数分解;(2) 将所有指数为偶数的整数归为一组,写成 \( {N}^{2} \) 的形式;(3) 取出所有剩余的整数;(4) 计算 \( N \) 的因数个数。
为立方数的约数个数
步骤:(1)对给定数进行质因数分解(Prime factorization)。(2)将所有指数为奇数且为3的倍数的整数归为一组,并以 \( {N}^{3} \) 的形式写出。(3)取出所有剩余的整数。(4)计算 \( N \) 的因数个数。
因数之和
因数之和为: \( \sigma \left( n\right) = \left( \frac{{p}_{1}^{a + 1} - 1}{{p}_{1} - 1}\right) \left( \frac{{p}_{2}^{b + 1} - 1}{{p}_{2} - 1}\right) \ldots \left( \frac{{p}_{k}^{m + 1} - 1}{{p}_{k} - 1}\right) \) (24.2)
由于我们已知 \( {a}^{n} - 1 = \left( {a - 1}\right) \left( {{a}^{n - 1} + {a}^{n - 2} + \ldots + {a}^{0}}\right) \)
因此 \( \sigma \left( n\right) = \left( {{p}_{1}^{a} + {p}_{1}^{a - 1} + \ldots + {p}_{1}^{0}}\right) \left( {{p}_{2}^{b} + {p}_{2}^{b - 1} + \ldots + {p}_{2}^{0}}\right) \ldots \left( {{p}_{k}^{m} + {p}_{k}^{m - 1} + \ldots + {p}_{k}^{0}}\right) \)
因数之积
因数之积为: \( \prod \left( n\right) = {\left( {p}_{1}^{a}{p}_{2}^{b}{p}_{3}^{c}\ldots {p}_{k}^{m}\right) }^{\frac{\left( {a + 1}\right) \left( {b + 1}\right) \left( {c + 1}\right) \cdots \left( {m + 1}\right) }{2}} \) (24.3)
1.4. 计数数字1到100的因数表
因数个数 | 计数数 | 计数数的个数 | 性质 |
1 | 1 | 1 | 平方数 |
2 | 2,3,5,7,11,13,17,19,23,29,31,37,41, 43, 47, 53, 59, 61, 67, 71, 73, 79, 83, 89, 97 | 25 | 质数 |
3 | 4,9,25,49 | 4 | 质数的平方 |
4 | 6,8,10,14,15,21,22,26,27,33,34,35, 38,39,46,51,55,57,58,62,65,69,74,77, 82,85,86,87,91,93,94,95 | 32 | |
5 | 16, 81 | 2 | 平方数 |
6 | 12,18,20,28,32,44,45,50,52,63,68,75, 76, 92, 98, 99 | 16 | |
7 | 64 | 1 | 平方数 |
8 | 24,30,40,42,54,56,66,70,78,88 | 10 | |
9 | 36,100 | 2 | 平方数 |
10 | 48,80 | 2 | |
11 | 0 | ||
12 | 60,72,84,90,96 | 5 |
2. 示例
2.1 因数、平方与立方
例1. \( {\left( {60}\right) }^{4} \) 的正整数真因子(不含1和 \( {\left( {60}\right) }^{4} \) 本身)的个数为
(A) 225 (B) 45 (D) 130 (E) 以上都不是
答案:(C)。 \( {5}^{4} \) 。
根据(24.1),除数个数为 \( \left( {8 + 1}\right) \left( {4 + 1}\right) \left( {4 + 1}\right) = {225} \) 。
除1和 \( {\left( {60}\right) }^{4} \) 外,共有223个因数。
例2. 设 \( N = {1000}^{2} + {1000} + {1001} \) 。有多少个正整数是 \( N \) 的因数?
(A) 6 (B) 8 (C) 15 (D) 24 (E) 27
解:(E)。
\( N = = {1000}^{2} + {1000} + {1001} = {1000}^{2} + {2000} + 1 = {1000}^{2} + 2 \times {1000} + 1 = ({1000} + \) \( 1{)}^{2} = {\left( {1001}\right) }^{2} = {\left( 7 \times {11} \times {13}\right) }^{2} = = {7}^{2} \times {11}^{2} \times {13}^{2}. \)
根据(24.1),因子数为 \( \left( {2 + 1}\right) \left( {2 + 1}\right) \left( {2 + 1}\right) = {27} \) 。
例3. \( {2}^{k} \) 是2016的一个因数,求 \( {k}^{2} \) 的最大可能值。
(A) 8 (B) 16 (C) 32 (D) 25 (E) 64
答案:(D)。
\( {2016} = {32} \times {63} = {2}^{5} \times {3}^{2} \times 7. \)
\( k = 5 \Rightarrow \;{k}^{2} = {25}. \)
例4. 设 \( m \) 和 \( n \) 为正整数,且满足 \( {150m} = {n}^{3} \) 。求 \( m + n \) 的最小可能值。
(A) 15 (B) 30 (C) 60 (D) 210 (E) 520
解答:(D)。
\( {150m} = {n}^{3}\; \Rightarrow \;2 \times 3 \times {5}^{2} \times m = {n}^{3}. \)
\( m \) 的最小值必须是 \( {2}^{2} \times {3}^{2} \times 5 = {180} \) 。
于是 \( {n}^{3} = {150m} = {150} \times {180} = {3}^{3} \times {10}^{3}\; \Rightarrow \;n = {30} \) 。
答案是 \( {180} + {30} = {210} \) 。
例5. 使得 \( {2016x} = {N}^{3} \) 成立的最小正整数 \( x \) ,其中 \( N \) 为整数,是
(A) 105 (B) 126 (C) 122 (D) 294 (E) 441
解答:(D)。
\( {2016} = {2}^{5} \times {3}^{2} \times 7 \) ,且仅当其所有不同质因数的指数均为3时,它才是一个立方数。因此, \( x = 2 \times 3 \times {7}^{2} = {294} \) 是使 \( {2016x} \) 为立方数的最小值。
例6. (2003 AMC 10 A) 随机抽取60的一个正因数,该因数小于7的概率是多少?
(A) \( \frac{1}{10} \) (B) \( \frac{1}{6} \) (C) \( \frac{1}{4} \) (D) \( \frac{1}{3} \) (E) \( \frac{1}{2} \)
解答:(E)。
方法1(官方解答):
60的因数有1,2,3,4,5,6,10,12,15,20,30,60。十二个因数中有六个小于7,因此概率为 \( \frac{1}{2} \) 。
方法2(我们的解答):
\( {60} = {2}^{2} \cdot 3 \cdot 5 \)
由(24.1)可知,因数的个数为 \( \left( {2 + 1}\right) \left( {1 + 1}\right) \left( {1 + 1}\right) = {12} \) 。
我们注意到这些因数中,1,2,3,4,5,6均小于7,因此概率为 \( \frac{1}{2} \) 。例7. 某数的质因数分解为 \( {2}^{2} \cdot {3}^{2} \cdot 5 \) 。它的正因数中有多少个是完全平方数?
(A) 1 (B) 2 (C) 3 (D) 4 (E) 5
解答:(D)。
\( {2}^{2} \cdot {3}^{2} \cdot 5 = {\left( 2 \cdot 3\right) }^{2} \cdot 5 \) .
\( \left( {2 \cdot 3}\right) \) 的任何因数都是完全平方数(perfect square)的因数。
答案是 \( \left( {1 + 1}\right) \left( {1 + 1}\right) = 4 \) 。
例8. \( {2}^{4} \times {3}^{6} \times {5}^{10} \) 有多少个奇完全平方因数?
(A) 24 (B) 6 (C) 12 (D) 14 (E) 18
解答:(A)。
\( {2}^{4} \times {3}^{6} \times {5}^{10} \Rightarrow {3}^{6} \times {5}^{10} \Rightarrow {\left( {3}^{3} \times {5}^{5}\right) }^{2} \)
奇完全平方因数的个数与 \( {3}^{3} \times {5}^{5} \) 的因数个数相同,可计算为 \( \left( {3 + 1}\right) \left( {5 + 1}\right) = {24} \) 。
例9. (AMC 12 A 2003 第23题) 乘积 \( 1! \cdot 2! \cdot 3! \cdot 4! \cdot 5! \cdot 6! \cdot 7! \cdot 8! \cdot 9! \) 有多少个完全平方因数?
(A) 504 (B) 672 (C) 864 (D) 936 (E) 1008
解答:(B)。
方法1(官方解答):
我们有
\( 1! \cdot 2! \cdot 3!\cdots 9! = \left( 1\right) \left( {1 \cdot 2}\right) \left( {1 \cdot 2 \cdot 3}\right) \cdots \left( {1 \cdot 2\cdots 9}\right) \)
\( = {1}^{9}{2}^{8}{3}^{7}{4}^{6}{5}^{5}{6}^{4}{7}^{3}{8}^{2}{9}^{1} = {2}^{30}{3}^{13}{5}^{5}{7}^{3} \) 。该乘积的完全平方因数形如 \( {2}^{2a}{3}^{2b}{5}^{2c}{7}^{2d} \) ,其中 \( 0 \leq a \leq {15},0 \leq b \leq 6,0 \leq c \leq 2 \) ,且 \( 0 \leq \) \( d \leq 1 \) 。因此共有 (16)(7)(3)(2) = 672 个这样的数。
方法2(我们的解答):
\( 1! \times 2! \times 3! \times 4! \times 5! \times 6! \times 7! \times 8! \times 9! \)
\( = 1 \times 2 \times \left( {3 \times 2}\right) \times \left( {4 \times 3 \times 2}\right) \times \left( {5 \times 4 \times 3 \times 2}\right) \times \left( {6 \times 5 \times 4 \times 3 \times 2}\right) \times (7 \times 6 \times 5 \times 4 \times 3 \)
\[ \times 2) \times \left( {8 \times 7 \times 6 \times 5 \times 4 \times 3 \times 2}\right) \times \left( {9 \times 8 \times 7 \times 6 \times 5 \times 4 \times 3 \times 2}\right) \]
\( = {2}^{30} \times {3}^{13} \times {5}^{5} \times {7}^{3} \)
\( = \left( {{2}^{30} \times {3}^{12} \times {5}^{4} \times {7}^{2}}\right) \times {3}^{1} \times {5}^{1} \times {7}^{1} \)
\( = {\left( {2}^{15} \times {3}^{6} \times {5}^{2} \times {7}^{1}\right) }^{2} \times {3}^{1} \times {5}^{1} \times {7}^{1} \)
完全平方因数的个数 \( = \left( {{15} + 1}\right) \left( {6 + 1}\right) \left( {2 + 1}\right) \left( {1 + 1}\right) = {672} \) 。
例10.(1985 AMC 12)设 \( p, q \) 和 \( r \) 为不同的质数,其中1不被视为质数。下列哪一项是 \( n = p{q}^{2}{r}^{4} \) 的最小正因子的正完全立方数?
(A) \( {p}^{8}{q}^{8}{r}^{8} \) (B) \( {\left( p{q}^{2}{r}^{2}\right) }^{3} \) (C) \( {\left( {p}^{2}{q}^{2}{r}^{2}\right) }^{3} \) (D) \( {\left( pq{r}^{2}\right) }^{3} \) (E) \( 4{p}^{3}{q}^{3}{r}^{3} \)
解答:(D)。
方法1(官方解答):
若 \( n = p{q}^{2}{r}^{4} \) 是立方数 \( c \) 的因子,则 \( c \) 的质因数分解中必须包含 \( p, q \) 和 \( r \) 。此外, \( p, q \) 和 \( r \) 在分解中的指数必须是3的倍数,且分别至少为1、2和4( \( (1,2,4 \) 为 \( p, q, r \) 在 \( n \) 中的指数)。因此 \( {p}^{3}{q}^{3}{r}^{6} = {\left( pq{r}^{2}\right) }^{3} \) 是最小的这样的立方数。
方法2(我们的解答):
我们排除(A)和(E),因为它们不是完全立方数。将(A)、(B)、(C)、(D)分别除以 \( n \) ,得到 \( {p}^{2}{q}^{4}{r}^{4},{p}^{5}{q}^{4}{r}^{2} \) 和 \( {p}^{2}q{r}^{2} \) 。可见(D)是最小的这样的立方数。
例11.(2005 AMC 10 A)有多少个正立方数能整除3! \( \cdot 5! \cdot 7!? \) (A) 2 (B) 3 (C) 4 (E) 6 解答:(E)。方法1(官方解答):写成质因数乘积形式,我们有 \( 3! \cdot 5! \cdot 7! = {2}^{8} \cdot {3}^{4} \cdot {5}^{2} \cdot 7 \) 。一个作为因子的立方数的质因数分解形式为 \( {2}^{\mathrm{p}} \cdot {3}^{\mathrm{q}} \cdot {5}^{\mathrm{r}} \cdot {7}^{\mathrm{s}} \) ,其中 \( p, q, r \) 和 \( s \) 均为3的倍数。 \( p \) 有3种可能取值:0、3和6。 \( q \) 有2种可能取值:0和3。 \( \mathrm{r} \) 和 \( \mathrm{s} \) 的唯一取值为0。因此共有 \( 6 = 3 \cdot 2 \cdot 1 \cdot 1 \) 个不同的立方数能整除 \( 3! \cdot 5! \cdot 7! \) 。
它们是 \( 1 = {2}^{0}{3}^{0}{5}^{0}{7}^{0},8 = {2}^{3}{3}^{0}{5}^{0}{7}^{0},{27} = {2}^{0}{3}^{3}{5}^{0}{7}^{0},{64} = {2}^{6}{3}^{0}{5}^{0}{7}^{0},{216} = {2}^{3}{3}^{3}{5}^{0}{7}^{0} \) ,
和 \( {1728} = {2}^{6}{3}^{3}{5}^{0}{7}^{0} \) 。
方法2(我们的解答):
我们有 \( 3! \cdot 5! \cdot 7! = {2}^{8} \cdot {3}^{4} \cdot {5}^{2} \cdot 7 \) 。
\( = {\left( {2}^{2} \times 3\right) }^{3} \times {2}^{2} \times {3}^{1} \times {5}^{2} \times {7}^{1} \)
我们只需计算 \( \left( {{2}^{2} \times 3}\right) \) 的因数个数,即 \( \left( {2 + 1}\right) (1 + \) 1) \( = 6 \) 。因此,完全立方因数的个数为6。
例12. 36的正整数约数之和是多少?
(A) 82 (B) 83 (C) 85 (D) 87 (E) 91
解:(E)。
\( {36} = 6 \times 6 = {2}^{2} \times {3}^{2} \)
所有除数之和:
\[ \sigma \left( n\right) = \left( {{p}_{1}^{a} + {p}_{1}^{a - 1} + \ldots + {p}_{1}^{0}}\right) \left( {{p}_{2}^{b} + {p}_{2}^{b - 1} + \ldots + {p}_{2}^{0}}\right) \]
\[ = \left( {{2}^{2} + {2}^{1} + {2}^{0}}\right) \left( {{3}^{2} + {3}^{1} + {3}^{0}}\right) = \left( {4 + 2 + 1}\right) \left( {9 + 3 + 1}\right) = 7 \times {13} = {91} \]
2.2 因子与整数
例13. 求恰有八个正整数因子的最小正整数。
(A) 24 (B) 30 (C) 40 (D) 54 (E) 22
解:(A)。
8可以写成2、2、2的乘积,也可以写成2和4的乘积。因此, \( 2 \times 3 \times 5 = {30} \) 和 \( {2}^{3} \times 3 = {24} \) 各有8个因数。其中最小的是24。
例14. 求所有恰好有12个正因数的两位正整数之和。
(A) 424 (B) 430 (C) 432 (D) 402 (E) 422
答案:(D)。
由于 \( {12} = 4 \times 3 = \left( {3 + 1}\right) \left( {2 + 1}\right) \) ,我们有: \( {2}^{3} \times {3}^{2} = {72} \) 。
\( {12} = 6 \times 2 = \left( {5 + 1}\right) \left( {1 + 1}\right) \) ,由此我们得到96。
\( {12} = 2 \times 2 \times 3 = \left( {1 + 1}\right) \left( {1 + 1}\right) \left( {2 + 1}\right) \) 产生 \( {2}^{2} \times 3 \times 5 = {60},{2}^{2} \times 3 \times 7 = {84} \) ,并且
\( {3}^{2} \times 2 \times 5 = {90} \) . 答案是 \( {72} + {96} + {60} + {84} + {90} = {402} \) .
例15. (2005 AMC 12 B 第21题) 一个正整数 \( n \) 有60个正约数,而 \( {7n} \) 有80个正约数。求最大的整数 \( k \) ,使得 \( {7}^{k} \) 整除 \( n \) 。
(A) 0 (B) 1 (C) 2 (D) 3 (E) 4
解答:(C)。
方法1(官方解答):
设 \( n = {7}^{k}Q \) ,其中 \( Q \) 为不含质因数7的素数乘积。令 \( d \) 为 \( Q \) 的正约数个数,则 \( n \) 有 \( \left( {k + 1}\right) d \) 个正约数。又 \( {7n} = {7}^{k + 1}Q \) ,故 \( {7n} \) 有 \( \left( {k + 2}\right) d \) 个正约数。于是 \( \left\lbrack {\left( {k + 2}\right) d}\right\rbrack /\left\lbrack {\left( {k + 1}\right) d}\right\rbrack = {80}/{60} = 4/3 \) 且 \( 3\left( {k + 2}\right) = 4(k + \) 1)。
因此 \( k = 2 \) 。注意 \( n = {2}^{19}{7}^{2} \) 满足题设条件。
方法2(我们的解法):
我们将60和80分解为尽可能接近的因数。
\( {60} = 3 \times 4 \times 5 \)
\( {80} = 4 \times 4 \times 5 \)
由于 \( n \) 有60个正约数,我们写成 \( n = {7}^{2} \times {2}^{3} \times {3}^{4} \) 。
由于 \( {7n} \) 有80个正约数,我们写成 \( {7n} = 7 \times {7}^{2} \times {2}^{3} \times {3}^{4} = {7}^{3} \times {2}^{3} \times {3}^{4} \) 。当 \( n \) 被 \( {7}^{k} \) 除时,得到 \( {7}^{2} \times {2}^{3} \times {3}^{4}/{7}^{k} \) 。于是 \( k = 2 \) 且 \( n = {7}^{2} \times {2}^{3} \times {3}^{4} \) 满足题设条件。
例16. (AIME 1988) 设 \( m/n \) 为随机选取 \( {10}^{99} \) 的一个正约数时,该约数是 \( {10}^{88} \) 的整数倍的概率,且 \( m/n \) 为最简分数。求 \( m + n \) 。
解答:634。
方法1(官方解答):
\( {10}^{99} \) 的正约数形如 \( {2}^{a} \cdot {5}^{b} \) ,其中 \( a \) 与 \( b \) 为整数,且 \( 0 \leq a \leq \) 99 且 \( 0 \leq b \leq {99} \) 。由于 \( a \) 与 \( b,{10}^{99} \) 各有100种取值, \( {100} \cdot {100} \) 共有 \( {100} \cdot {100} \) 个正整数约数。其中, \( {10}^{88} = {2}^{88} \cdot {5}^{88} \) 的倍数必须
满足不等式 \( {88} \leq a \leq {99} \) 和 \( {88} \leq b \leq {99} \) 。因此 \( a \) 和 \( b \) 各有12种选择;即 \( {10}^{99} \) 的 \( {100} \cdot {100} \) 个约数中, \( {12} \cdot {12} \) 是 \( {10}^{88} \) 的倍数。于是所求概率为 \( \mathrm{m}\frac{m}{n} = \frac{{12} \cdot {12}}{{100} \cdot {100}} = \frac{9}{625} \) 和 \( m + n = {634} \) 。
方法二(我们的解法):
\( {10}^{99} = {2}^{99} \cdot {5}^{99} \) 。根据(24.1), \( {10}^{99} \) 的约数个数为 \( \left( {{99} + 1}\right) \left( {{99} + 1}\right) = {100} \) \( \times {100} \) 。这将作为我们分数的分母。
\( {10}^{99} = {10}^{{88} + {11}} = {10}^{88}\left( {10}^{11}\right) \) 。是 \( {10}^{88} \) 的倍数的约数个数等于 \( \left( {10}^{11}\right) \) 的因数个数。由于 \( {10}^{11} = {2}^{11} \times {5}^{11} \) ,因数个数为 \( \left( {{12} \times {12}}\right) \) 。
随机选取 \( {10}^{99} \) 的一个正约数,它是 \( {10}^{88} \) 的整数倍的概率为 \( \frac{m}{n} = \frac{{12} \cdot {12}}{{100} \cdot {100}} = \frac{9}{625} \) 和 \( m + n = {634} \) 。
2.3. 可约或不可约分数
例17. 有多少个正整数 \( n \) 使得 \( \frac{72}{n} \) 也是整数?
(A) 16 (B) 12 (C) 8 (D) 4 (E) 3
解答:(B)。
\( {72} = 8 \times 9 = {2}^{3} \times {3}^{2} \) 有 \( \left( {3 + 1}\right) \left( {2 + 1}\right) = {12} \) 个因数, \( n \) 必须是其中之一才能使 \( \frac{72}{n} \) 为整数,因此答案是12。
例18. 有多少个自然数 \( n \) 使得 \( \frac{36}{{2n} + 1} \) 为自然数?
(A) 9 (B) 8 (C) 4 (D) 2 (E) 1
解答:(D)。
\( {36} = {6}^{2} = {\left( 2 \times 3\right) }^{2} = {2}^{2} \times {3}^{2} \) 有 \( 3 \times 3 = 9 \) 个因数, \( {2n} + 1 \) 必须是这9个因数之一才能使 \( \frac{36}{{2n} + 1} \) 为自然数。注意由于 \( n \) 为正整数, \( {2n} + 1 \) 必须为奇数。在这9个因数中,有三个奇因数:1、
3和9。于是 \( {2n} + 1 = 1, n = 0 \) (因 \( n \) 为正而被忽略); \( {2n} + 1 = 3, n = 1 \) ;以及 \( {2n} + 1 = 9, n = 4 \) 。答案为(D)。
例19.(1985 AMC 12 第26题)求最小的正整数 \( n \) ,使得 \( \frac{n - {13}}{{5n} + 6} \) 为非零可约分数。
(A) 45 (B) 68 (C) 155 (D) 226 (E) 无以上选项
解答:(E)。
方法1(官方解答):
\( \frac{n - {13}}{{5n} + 6} \) 可约且非零当且仅当其倒数 \( \frac{n - {13}}{{5n} + 6} \) 存在且可约。
通过长除法, \( \frac{n - {13}}{{5n} + 6} = 5 + \frac{71}{n - {13}} \) 。因此必要且充分的条件是 \( \frac{71}{n - {13}} \) 可约。由于71为素数, \( n - {13} = {71} \) 或 \( n = {84} \) 即为最小解。
方法2(我们的解法):
若 \( \frac{n - {13}}{{5n} + 6} \) 可约,则存在某个素数 \( p \) 同时整除 \( n - {13} \) 与 \( {5n} + 6 \) 。
根据定理2,该 \( p \) 亦将整除这些项的任意线性组合,于是 \( p \) 整除 \( j\left( {{5n} + 6}\right) + k\left( {n - {13}}\right) \) 。若令 \( j = 1 \) 与 \( k = - 5 \) (以消去 \( n) \) ),则得 \( j\left( {{5n} + 6}\right) + k\left( {n - {13}}\right) = \left( {{5n} + 6}\right) - 5\left( {n - {13}}\right) = {71} \) 。因此 \( p = {71} \) 且 \( n - \) \( {13} = {71k} \) 对某个整数 \( k \) 成立。由于 \( n > 0 \) 且 \( n - {13} \neq 0 \) , \( p \) 的最小值在 \( k = 1 \) 时取得,故 \( n - {13} = {71} \Rightarrow n = {71} + {13} = {84} \) 。
例20.(1959年国际数学奥林匹克)证明:对任意自然数 \( n \) ,分数 \( \frac{{21n} + 4}{{14n} + 3} \) 不可约。解法:方法1:
若 \( \frac{{21n} + 4}{{14n} + 3} \) 可约,则存在某个公因子 \( m \) 同时整除 \( {21n} \) \( + 4 \) 与 \( {14n} + 3 \) 。
根据定理2, \( m \) 亦整除 \( j\left( {{14n} + 3}\right) + k\left( {{21n} + 4}\right) \) 。若令 \( j = 3 \) 与 \( k = - 2 \) (以消去 \( n \) ),则得 \( j\left( {{14n} + 3}\right) + k\left( {{21n} + 4}\right) = 3\left( {{14n} + 3}\right) - 2({21n} + \) \( 4) = 1 \) 。于是 \( {21n} + 4 \) 与 \( {14n} + 3 \) 互素,故分数 \( \frac{{21n} + 4}{{14n} + 3} \) 对任意自然数 \( n \) 均不可约。
方法2:
\( \frac{{21n} + 4}{{14n} + 3} = 1 + \frac{{7n} + 1}{{14n} + 3} \)
若 \( \frac{{21n} + 4}{{14n} + 3} \) 可约,则存在某个公因子 \( m \) 同时整除 \( {7n} \) \( + 1 \) 与 \( {14n} + 3 \) 。
根据定理2, \( m \) 亦整除 \( j\left( {{14n} + 3}\right) + k\left( {{7n} + 1}\right) \) 。若令 \( j = 1 \) 与 \( k = - 2 \) (以消去 \( n \) ),则得 \( j\left( {{14n} + 3}\right) + k\left( {{7n} + 1}\right) = {14n} + 3 - 2\left( {{7n} + 1}\right) = \) 1。于是 \( {7n} + 1 \) 与 \( {14n} + 3 \) 互素,故分数 \( \frac{{21n} + 4}{{14n} + 3} \) 对任意自然数 \( n \) 均不可约。
例21. 求 \( x \) 的最大整数值,使得 \( \frac{{x}^{2} + {2x} - {15}}{x - 5} \) 为整数。
(A) 45 (B) 68 (C) 15 (D) 25 (E) 以上皆非
解答:(D)。
我们将 \( \frac{{x}^{2} + {2x} - {15}}{x - 5} \) 写作 \( \frac{{x}^{2} + {2x} - {15}}{x - 5} = \frac{\left( {x - 5}\right) \left( {x + 7}\right) + {20}}{x - 5} = x + 7 + \frac{20}{x - 5} \) 。
\( x - 5 \) 必须是20的因数。20是20的最大因数,而我们要找最大的整数值,因此 \( x - 5 = {20} \) 和 \( x = {25} \) 。
例22.(2002 AMC 10B)有多少个整数 \( n \) 使得 \( \frac{n}{{20} - n} \) 是一个整数的平方?
(A) 1 (B) 2 (C) 3 (D) 4 (E) 10
解答:(D)。
方法1(官方解答):
假设 \( \frac{n}{{20} - n} = {k}^{2} \) 对某个 \( k \geq 0 \) 成立,则 \( n = \frac{{20}{k}^{2}}{{k}^{2} + 1} \) 。
由于 \( {k}^{2} \) 与 \( {k}^{2} + 1 \) 互质且 \( n \) 为整数, \( {k}^{2} + \) 1必须是20的因数。这只在 \( k = 0,1,2 \) 或3时成立。对应的 \( n \) 值为0,10,16和18。
方法2(我们的解答):
若 \( \frac{n}{{20} - n} = {m}^{2} \) 对某个整数 \( m \geq 0 \) 成立,则
\( n = \frac{{20}{m}^{2}}{{m}^{2} + 1} = \frac{{20}{m}^{2} + {20} - {20}}{{m}^{2} + 1} = {20} - \frac{20}{{m}^{2} + 1}. \)
由于 \( n \) 为整数, \( {m}^{2} + 1 \) 必须是 \( {20} = {2}^{2} \times 5 \) 的因数。
\( {m}^{2} + 1 = 1\; \Rightarrow \;m = 0 \Rightarrow \;n = 0 \) 和 \( \frac{n}{{20} - n} = 0. \)
\( {m}^{2} + 1 = 2\; \Rightarrow \;m = 1 \) 或 \( m = - 1\; \Rightarrow \;n = {10} \) 和 \( \frac{n}{{20} - n} = 1 \) 。
\( {m}^{2} + 1 = 4\; \) ( \( m \) 无整数解)。
\[ {m}^{2} + 1 = 5\; \Rightarrow \;m = 2\text{or}m = - 2\; \Rightarrow \;n = {16}\text{and}\frac{n}{{20} - n} = 4\text{.} \]
\[ {m}^{2} + 1 = {10} \Rightarrow \;m = 3\text{ or }m = - 3\; \Rightarrow \;n = {18}\text{ and }\frac{n}{{20} - n} = 1. \]
\( {m}^{2} + 1 = {20}\; \) ( \( m \) 无整数解)。
\( n \) 的取值为0,10,16和18。
例23.(ARML)恰有五个正整数 \( n \) 使得 \( \frac{{\left( n + 1\right) }^{2}}{n + {23}} \) 为整数。求最大的 \( n \) 。答案:461。
将 \( {\left( n + 1\right) }^{2} \) 除以 \( n + {23} \) 得到 \( n - {21} + \frac{484}{n + {23}} \) 。使该式为整数的最大 \( n \) 来自 \( n + {23} = {484} \) 。于是 \( n = {461} \) 。
例24.有多少个正整数 \( n \) 使得 \( \frac{{2n} + 1}{{2n}\left( {n + 1}\right) } \) 为非零可约分数?
(A)1(B)2(C)3(D)4(E)0 答案:(E)。我们考察 \( {2n} + 1 \) 和 \( n + 1 \) 两项。若 \( \frac{{2n} + 1}{n + 1} \) 可约,则存在某个公因子 \( m \) 同时整除 \( {2n} + \) 1和 \( n + 1 \) 。
根据定理2, \( m \) 也整除 \( j\left( {n + 1}\right) + k\left( {{2n} + 1}\right) \) 。若令 \( j = 2 \) 和 \( k = - 1 \) (以消去 \( n \) ),则得 \( 2\left( {n + 1}\right) - \left( {{2n} + 1}\right) = 1 \) 。因此 \( {2n} + 1 \) 与 \( n + 1 \) 互质。
现在我们考察 \( {2n} + 1 \) 和 \( {2n} \) 两项。
若 \( \frac{{2n} + 1}{2n} \) 可约,则存在某个公因子 \( m \) 同时整除 \( {2n} + \)
1和 \( n \) 。
根据定理2, \( m \) 也整除 \( j\left( {{2n} + 1}\right) + k\left( {2n}\right) \) 。若令 \( j = 1 \) 和 \( k = - 1 \) (以消去 \( n \) ),则得 \( {2n} + 1 - {2n} = 1 \) 。因此 \( {2n} + 1 \) 与 \( n \) 也互质。
因此不存在任何整数 \( n \) 使得 \( \frac{{2n} + 1}{{2n}\left( {n + 1}\right) } \) 为非零可约分数。
例25.(AIME)若 \( x \) 和 \( y \) 为整数且满足 \( {y}^{2} + 3{x}^{2}{y}^{2} = {30}{x}^{2} \) +517,求 \( 3{x}^{2}{y}^{2} \) 。
答案:588。
方法1(官方解答):
将给定方程改写为 \( \left( {{y}^{2} - {10}}\right) \left( {3{x}^{2} + 1}\right) = 3 \cdot {13}^{2} \) 的形式,并注意,由于 \( y \) 是整数且 \( 3{x}^{2} + 1 \) 为正整数, \( {y}^{2} - {10} \) 必为正整数。因此 \( {y}^{2} - {10} = 1,3,{13},{39},{169} \) 或507,这意味着 \( {y}^{2} = {11} \) 为13、23、49、179或517。由于第二个列表中唯一的完全平方数是49,于是 \( {y}^{2} - {10} = {39} \) ,从而 \( 3{x}^{2} + 1 = {13},{x}^{2} = 4 \) 且 \( 3{x}^{2}{y}^{2} = {12} \cdot {49} = \) 588。
方法二(我们的解法):
\[ {y}^{2} = \frac{{30}{x}^{2} + {517}}{3{x}^{2} + 1} = \frac{{30}{x}^{2} + {10} + {507}}{3{x}^{2} + 1} = {10} + \frac{507}{3{x}^{2} + 1} = {10} + \frac{3 \times {13}^{2}}{3{x}^{2} + 1} \]
我们知道 \( 3{x}^{2} + 1 \geq 4 \) ,因此 \( 3{x}^{2} + 1 \) 可以取以下值: \( {13},3 \times {13},{13}^{2} \) 、 \( 3 \times {13}^{2} \) 。分别求出每种情况下的 \( x \) ,只有整数解为 \( x = 2 \) 和 \( x = 4 \) 。当 \( x = 2 \) 时, \( y = 7 \) 的值;当 \( x = 4 \) 时, \( y \) 无整数值。因此 \( 3{x}^{2}{y}^{2} = {588} \) 。
例26.(AIME)最大的正整数 \( n \) 是多少,使得 \( {n}^{3} + {100} \) 能被 \( n + {10} \) 整除?
解答:890。
方法一(官方解法):
通过除法得到 \( {n}^{3} + {100} = \left( {n + {10}}\right) \left( {{n}^{2} - {10n} + {100}}\right) - {900} \) 。因此,若 \( n + {10} \) 整除 \( {n}^{3} + {100} \) ,则它也必须整除900。此外,由于 \( n \) 在 \( n + {10} \) 最大时取最大值,而900的最大因数是900,我们必有 \( n + \) \( {10} = {900} \) 。因此, \( n = {890} \) 。
方法二(我们的解法):
\[ \frac{{n}^{3} + {100}}{n + {10}} = \frac{{n}^{3} + {1000} - {900}}{n + {10}} = \frac{{n}^{3} + {10}^{3} - {900}}{n + {10}}. \]
\[ = \frac{{n}^{3} + {10}^{3}}{n + {10}} - \frac{900}{n + {10}} = {n}^{2} - {10n} + {100} - \frac{900}{n + {10}}. \]
我们知道 \( \frac{{n}^{3} + {100}}{n + {10}} \) 是正整数,因此 \( \frac{900}{n + {10}} \) 必须是整数,且900必须能被 \( n + {10} \) 整除。
为了得到最大的正整数 \( n \) ,我们令 \( n + {10} = {900} \) 。因此, \( n = \) 890。
习题
问题1. 下列乘积共有多少个不同的正因数:(12)(15)(17)?
(A) 9 (B) 6 (C) 18 (D) 36 (E) 56
问题2. 设 \( N = {71}^{3} + 3 \times {71}^{2} + 3 \times {71} + 1 \) 。有多少个正整数是 \( N \) 的因数?
(A) 54 (B) 45 (C) 60 (D) 69 (E) 70
问题3. \( {2}^{N} \) 是20!的因数。求 \( N \) 的最大可能值。
(A) 18 (B) 16 (C) 32 (D) 33 (E) 12
问题4. 求最小的正整数,使得252乘以该数后结果为完全立方数。
(A) 2352 (B) 147 (C) 294 (D) 333 (E) 128
问题5. 求最小的正整数,使得180乘以该数后所得乘积为完全立方数。
(A) 50 (B) 75 (C) 150 (D) 180 (E) 5
问题6. 若从80的所有正因数中随机选取一个,求所选因数小于10的概率。
(A) \( \frac{3}{5} \) (B) \( \frac{5}{7} \) (C) \( \frac{1}{5} \) (D) \( \frac{2}{5} \) (E) \( \frac{7}{8} \)
问题7. 432的正整数因数中有多少个是完全平方数?
(A) 3 (B) 6 (C) 12 (D) 20 (E) 2
问题8. \( {2}^{4} \times {3}^{6} \times {5}^{10} \times {7}^{9} \) 有多少个奇完全平方因数?
(A) 60 (B) 120 (C) 30 (D) 115 (E) 20
问题9. 下列哪个数是完全平方数?
(B) \( {120}! \cdot {122} \) ! (C) 119! ·120! (D) 119! · 121! (E) \( {121}! \cdot {122} \) !
问题10. \( {2}^{4} \times {3}^{6} \times {5}^{10} \) 有多少个完全立方因数?
(A) 24 (B) 20 (C) 16 (D) 14 (E) 12
问题11. 乘积 \( 1! \times 2! \times 3! \times 4! \) \( \times 5! \times 6! \times 7! \times 8! \times 9!? \) 有多少个完全立方因数?
(A) 104 (B) 220 (C) 164 (D) 136 (E) 108
问题12. 12的所有正整数约数的乘积是多少?
(A) 1024 (B) 1220 (C) 1642 (D) 1716 (E) 1728
问题13. 求小于100且恰有4个不同正因子的正整数的个数。
(A) 24 (B) 30 (C) 32 (D) 34 (E) 22
问题14. 求小于200且恰有9个约数的所有数之和。
(A) 332 (B) 432 (C) 532 (D) 232 (E) 122
问题15.(1996 AMC 12 第29题)若 \( n \) 为正整数,且 \( {2n} \) 有28个正因数, \( {3n} \) 有30个正因数,则 \( {6n} \) 有多少个正因数?
(A) 32 (B) 34 (C) 35 (D) 36 (E) 38
问题16.(2000 AIME II)最小的正整数,其正因数中有六个正奇数和十二个正偶数,是多少?
问题17. 有多少个正整数 \( n \) 使得 \( \frac{60}{n} \) 也是整数?
(A) 16 (B) 12 (C) 8 (D) 4 (E) 3
问题18. 有多少个正整数 \( x \) 使得 \( \frac{36}{x + 3} \) 为整数?
(A) 9 (B) 8 (C) 6 (D) 4 (E) 3
问题19.(AMC)若 \( p \) 为正整数,则 \( \frac{{3p} + {25}}{{2p} - 5} \) 为正整数的充要条件是 \( p \) 取何值?(A) 至少为3 (B) 至少为3且不超过35 (C) 不超过35 (D) (E) 等于3或35
问题20. 小于50的正整数 \( n \) 中,有多少个使得 \( \frac{{4n} + 5}{{7n} + 6} \) 可为正整数?
(A) 0 (B) 2 (C) 3 (D) 4 (E) 超过4
问题21. 使得 \( \frac{{x}^{2} + {2x} + 5}{x - 3} \) 为整数的 \( x \) 的最大整数值是多少?
(A) 45 (B) 68 (C) 15 (D) 23 (E) 以上都不是
问题22. 有多少个整数 \( n \) 使得 \( \frac{n}{{40} - n} \) 为整数的平方?
(A) 1 (B) 2 (C) 3 (D) 4 (E) 10
问题23. 有多少个正整数 \( k \) 使得 \( {kx} - {72} = {3k} \) 对 \( x \) 有整数解?
(A) 10 (B) 12 (C) 13 (D) 14 (E) 6
问题24. 求最小的正整数 \( n \) ,使得 \( \frac{{3n} + 1}{n\left( {{2n} - 1}\right) } \) 为非零可约分数。
(A) 4 (B) 6 (C) 5 (D) 2 (E) 3
问题25. 求方程的整数解对数
\( {2xy} - 2{x}^{2} + {3x} - {5y} + {11} = 0. \)
(A) 0 (B) 2 (C) 3 (D) 4 (E) 超过4
问题26. 在1到100(含)的正整数中,有多少个 \( n \) 使得 \( \frac{{n}^{2} + 7}{n + 4} \) 为非零可约分数?
(A) 1 (B) 2 (C) 3 (D) 4 (E) 10
问题27. (2004 AIME 2 #8) 有多少个 \( {2004}^{2004} \) 的正整数约数恰好能被2004个正整数整除?
解答:
问题1. 解答:D。
对(12)(15)(17)进行质因数分解:
(12) \( \left( {15}\right) \left( {17}\right) = {189} = 3 \times {2}^{2} \times 3 \times 5 \times {17} = {2}^{2} \times {3}^{2} \times {5}^{1} \times {17}^{1} \)
约数个数 \( d = \left( {2 + 1}\right) \left( {2 + 1}\right) \left( {1 + 1}\right) \left( {1 + 1}\right) = {36} \) 。
问题2. 解答:(E)。
根据二项式定理, \( N = {\left( {71} + 1\right) }^{3} = {\left( {72}\right) }^{3} = {\left( {3}^{2} \times {2}^{3}\right) }^{3} = {3}^{6} \times {2}^{9} \) 。
由(24.1),因子个数为 \( \left( {6 + 1}\right) \left( {9 + 1}\right) = {70} \) 。
问题3. 解答:(A)。
\[ N = \left\lfloor \frac{20}{2}\right\rfloor + \left\lfloor \frac{20}{{2}^{2}}\right\rfloor + \left\lfloor \frac{20}{{2}^{3}}\right\rfloor + \left\lfloor \frac{20}{{2}^{4}}\right\rfloor = {10} + 5 + 2 + 1 = {18}. \]
问题4. 解答:(C)。
设 \( {m}^{3} \) 为完全立方数(perfect cube), \( n \) 为最小正整数。
\( {252} = {2}^{2} \times 7 \times {3}^{2} \)
\[ {252} \times n = {2}^{2} \times 7 \times {3}^{2} \times n = {m}^{3}. \]
\[ n = 2 \times 3 \times {7}^{2} = {294} \]
问题5。解答:(C)。
设 \( {m}^{3} \) 为完全立方数(perfect cube), \( n \) 为最小正整数。
\( {180} = {2}^{2} \cdot {3}^{2} \cdot 5 \) .
\( {180} \times n = {2}^{2} \cdot {3}^{2} \cdot 5 \times n = {m}^{3}. \)
\( n = 2 \cdot 3 \cdot {5}^{2} = {150} \) .
问题6。解答:(E)。
\( {80} = {2}^{4} \times 5 \) 共有 \( \left( {4 + 1}\right) \left( {1 + 1}\right) = {10} \) 个因数。
我们看到这些因数中,1、2、4、5、8均小于10,因此
概率为 \( \frac{1}{2} \) 。
问题7。解答:B。
\( {432} = {2}^{4} \cdot {3}^{3} = {\left( {2}^{2} \cdot 3\right) }^{2} \cdot 3. \) 任何 \( \left( {{2}^{2} \cdot 3}\right) \) 的因数都会是完全平方数(perfect square)的因数。答案是 \( \left( {2 + 1}\right) \left( {1 + 1}\right) = 6 \) 。
问题8。解答:(B)。
我们只需考察 \( {3}^{6} \times {5}^{10} \times {7}^{9} \) 的完全平方因数的个数。 \( {3}^{6} \times {5}^{10} \times {7}^{9} = {\left( {3}^{3} \times {5}^{5} \times {7}^{4}\right) }^{2} \times 7. \)
共有 \( \left( {3 + 1}\right) \left( {5 + 1}\right) \left( {4 + 1}\right) = {120} \) 个奇完全平方因数。
问题9。解答:(A)。
注意,对于 \( m < n \) 我们有 \( m! \cdot n! = \left( {m!}\right) 2 \cdot \left( {m + 1}\right) \cdot \left( {m + 2}\right) \cdots n \) 。因此 \( m! \cdot n! \) 是完全平方数当且仅当 \( \left( {m + 1}\right) \cdot \left( {m + 2}\right) \cdots \cdots n \) 是完全平方数。在五个选项中,该量分别为 \( {121},{121} \cdot {122},{120} \) 、 \( {120} \cdot {121} \) 和122,其中只有121是完全平方数。因此答案是 \( {120}! \cdot {121}! \) 。
问题10。解答:(A)。
\( {2}^{4} \times {3}^{6} \times {5}^{10} = {2}^{3} \times {\left( {3}^{2}\right) }^{3} \times {\left( {5}^{3}\right) }^{3} \times 2 \times 5 = {\left( {2}^{1} \times {3}^{2} \times {5}^{3}\right) }^{3} \times 2 \times 5. \)
我们只需计算 \( \left( {{2}^{1} \times {3}^{2} \times {5}^{3}}\right) \) 的因数个数,即 \( (1 + \) \( 1)\left( {2 + 1}\right) \left( {3 + 1}\right) = {24} \) 。因此完全立方因数的个数为24。
问题11。解答:(B)。
\[ 1! \times 2! \times 3! \times 4! \times 5! \times 6! \times 7! \times 8! \times 9! \]
\[ = {2}^{30} \times {3}^{13} \times {5}^{5} \times {7}^{3} \]
\[ = {\left( {2}^{10} \times {3}^{4} \times {5}^{1} \times {7}^{1}\right) }^{3} \times {3}^{1} \times {5}^{2} \]
我们只需计算 \( {2}^{10} \times {3}^{4} \times {5}^{1} \times {7}^{1} \) 的因数个数,即 \( = \) \( \left( {{10} + 1}\right) \left( {4 + 1}\right) \left( {1 + 1}\right) \left( {1 + 1}\right) = {220} \)
问题12。解答:(E)。
\( {12} = 3 \times {2}^{2} \) .
其所有因数之积为 \( = {\left( 3 \times {2}^{2}\right) }^{\left( {1 + 1}\right) \left( {2 + 1}\right) /2} = {12}^{3} = {1728} \) 。
问题13。解答:(C)。
由于4可以写成 \( 2 \times 2 = \left( {1 + 1}\right) \left( {1 + 1}\right) = \left( {3 + 1}\right) \) ,该数必须是两个素数之积或一个素数的立方(两种情况)。
小于100且能表示为两个素数之积的数有30个,小于100的素数立方有2个。因此共有32个这样的正整数。
问题14。解答:(A)。
要使一个数恰有9个因数,其素因数分解的指数只能是8,或2与2。 \( \left( {9 = 1 \times 9 = 3 \times 3}\right) \)
\( {2}^{8} = {256} \) 大于200,故可排除该情形。
\( {2}^{2} \times {3}^{2} = {36};{2}^{2} \times {5}^{2} = {100};{3}^{2} \times {5}^{2} = {225} \) (超过200); \( {2}^{2} \times {7}^{2} = {196} \) 。
总和为 \( {36} + {100} + {196} = {332} \) 。
问题15。解答:(C)。
方法1(官方解答):
设 \( {2}^{{e}_{1}}{3}^{{e}_{2}}{5}^{{e}_{3}}\cdots \) 为 \( n \) 的素因数分解,则 \( n \) 的正因数个数为 \( \left( {{e}_{1} + 1}\right) \left( {{e}_{2} + 1}\right) \left( {{e}_{3} + 1}\right) \cdots \) 。根据已知条件,有 \( {28} = \left( {{e}_{1} + 2}\right) \left( {{e}_{2} + 1}\right) P \) 和 \( {30} = \left( {{e}_{1} + 1}\right) \left( {{e}_{2} + 2}\right) P \) ,其中 \( P = \left( {{e}_{3} + 1}\right) \left( {{e}_{4} + 1}\right) \) 。
- . 用第二个方程减去第一个方程,我们得到 \( 2 = \left( {{e}_{1} - {e}_{2}}\right) P \) ,因此要么 \( {e}_{1} - {e}_{2} = 1 \) 且 \( P = 2 \) ,要么 \( {e}_{1} - {e}_{2} = 2 \) 且 \( P = 1 \) 。第一种情况给出 \( {14} = \left( {e}_{1}\right. \) \( + 2){e}_{1} \) 和 \( {\left( {e}_{1} + 1\right) }^{2} = {15} \) ;由于 \( {e}_{1} \) 是非负整数,这不可能。在第二种情况下, \( {e}_{2} = {e}_{1} - 2 \) 且 \( {30} = \left( {{e}_{1} + 1}\right) {e}_{1} \) ,由此我们得到 \( {e}_{1} = 5 \) 和 \( {e}_{2} = 3 \) 。于是 \( n = {2}^{5}{3}^{3} \) ,因此 \( {6n} = {2}^{6}{3}^{4} \) 有 \( \left( {6 + 1}\right) \left( {4 + 1}\right) = {35} \) 个正因数。
方法二(我们的解法):
我们将60和80分解因数,使所有因数尽可能接近。
\( {28} = 4 \times 7 \)
\( {30} = 5 \times 6 \) .
由于 \( {2n} \) 有28个因数,我们写成 \( n = {2}^{5} \times {3}^{3} \) 。于是 \( {2n} = 2 \times {2}^{5} \times {3}^{3} = {2}^{6} \times {3}^{3} \) (含 \( 7 \times 4 = {28} \) 个因数)。由于 \( {3n} \) 有30个因数,我们写成 \( {3n} = 3 \times {2}^{5} \times {3}^{3} = {2}^{5} \times {3}^{4} \) (含 \( 6 \times 5 = {30} \) 个因数)。因此 \( {6n} = 6 \times {2}^{5} \times {3}^{3} = {2}^{6} \times {3}^{4} \) 。根据(24.1), \( {2}^{6} \times {3}^{4} \) 的因数个数为 \( (6 + \) 1) \( \left( {4 + 1}\right) = {35} \) 。
问题16。答案:180。
方法一(官方解法):
对任意正整数 \( x \) ,设 \( x = {2}^{{a}_{1}}{p}_{2}{}^{{a}_{2}}{p}_{3}{}^{{a}_{3}}\cdots {p}_{n}{}^{{a}_{n}} \) 为其素因数分解,其中 \( 0 \leq {a}_{1} \) 且 \( 1 \leq {a}_{i} \) 对 \( 1 < i \) 成立,且 \( {p}_{i} \) 为素数,满足 \( 2 < {p}_{2} < \) \( {p}_{3} < \ldots < {p}_{n} \) 。若 \( x \) 具有所需因数个数,则
\( 6 = \mathop{\prod }\limits_{{i = 2}}^{n}\left( {{a}_{i} + 1}\right) \) 且 \( {12} = {a}_{1}\mathop{\prod }\limits_{{i = 2}}^{n}\left( {{a}_{i} + 1}\right) \) ,这意味着 \( {a}_{1} = 2 \) 。还可推出
要么 \( n = 2 \) 和 \( {a}_{2} = 5 \) ,要么 \( n = 3 \) 和 \( \left\{ {{a}_{2},{a}_{3}}\right\} = \{ 1,2\} \) 。因此 \( x = {2}^{2}{p}_{2}{}^{5} \) ,或 \( x = {2}^{2}{p}_{2}{}^{2}{p}_{3} \) ,或 \( x = {2}^{2}{p}_{2}{p}_{3}{}^{2} \) 。为了找到最小的 \( x \) ,测试 \( {p}_{2} \) 和 \( {p}_{3} \) 的最小可能值。这给出三个候选 \( {2}^{2}{3}^{5} = {972} \) 、 \( {2}^{2} \cdot {3}^{2} \cdot 5 = {180} \) 和 \( {2}^{2} \cdot 3 \cdot {5}^{2} = {300} \) ,其中 \( x = {180} \) 最小。
方法二(我们的解法):
6可写成 \( 3 \times 2 = \left( {2 + 1}\right) \left( {1 + 1}\right) \) ,12可写成 \( 3 \times 2 \times 2 = (2 + \) \( 1)\left( {1 + 1}\right) \left( {1 + 1}\right) \) 。
由于我们要找的是具有六个正奇因子的最小正整数,该数的两个因子就是最小的两个奇素数,即3和5。满足条件的最小数是 \( {3}^{2} \times {5}^{1} \cdot {3}^{2} \times {5}^{1} \) ,它将是该整数的因子。
现在我们已经覆盖了该数中的所有奇因子,需要确定它包含多少个偶素因子2,使得偶因子的总数为12。由于 \( 3 \times 2 \times 2 = \left( {2 + 1}\right) \left( {1 + 1}\right) \left( {1 + 1}\right) \) ,答案是 \( 2\left( {{2}^{1} \times {3}^{2} \times {5}^{1}}\right) = {180}. \) 。
问题17。解答:(B)。
\( {60} = 6 \times {10} = {2}^{2} \times 3 \times 5 \) 有12个因子。为了使 \( \frac{60}{n} \) 为整数, \( n \) 必须是其中之一。因此答案是12。
问题18。解答:(C)。
\( {36} = {6}^{2} = {\left( 2 \times 3\right) }^{2} = {2}^{2} \times {3}^{2} \) 有 \( 3 \times 3 = 9 \) 个因子。为了使 \( \frac{36}{x + 3} \) 为整数, \( x + 3 \) 必须是这9个因子之一。注意由于 \( x \) 是正整数,所以
\( x + 3 \geq 4 \) 。在这9个因子中,有3个小于4。因此答案是9 \( - 3 = 6 \) 。
问题19。解答:(B)。
方法一(官方解法):
设 \( \left( {{3p} + {25}}\right) /\left( {{2p} - 5}\right) = n \) 为正整数。则 \( {3p} + {25} = {kn},{2p} - 5 = k;\therefore \) \( k\left( {{2n} - 3}\right) = {65} = 1 \cdot {65} = 5 \cdot {13}. \)
\( \therefore k = 1,{65},5 \) ,或13和 \( {2n} - 3 = {65},1,{13} \) ,或相应地5。 \( \therefore {2p} = 5 + 1,5 \) \( + {65},5 + 5 \) ,或 \( 5 + {13};\therefore p = 3,{35},5 \) ,或9。
\( \therefore \) 选项(B)正确。
方法二(我们的解法):
若 \( \frac{{3p} + {25}}{{2p} - 5} \) 为正整数,则存在某个公因数 \( n \) 能整除
\( {3p} + {25} \) 和 \( {2p} - 5 \) 两者。但此时 \( n \) 整除这些项的任意线性组合,因此 \( n \) 整除 \( j\left( {{3p} + {25}}\right) + k\left( {{2p} - 5}\right) \) 。若令 \( j = 2 \) 和 \( k = - 3 \) (以消去 \( n \) ),我们得到 \( j\left( {{3p} + {25}}\right) + k\left( {{2p} - 5}\right) = 2\left( {{3p} + {25}}\right) - 3\left( {{2p} - 5}\right) = {65} = 1 \times {65} = 5 \) ×13。设 \( {2p} - 5 = 1;{2p} - 5 = {65};{2p} - 5 = 5 \) ;且 \( {2p} - 5 = {13} \) ,我们得到 \( p = 3,5,9 \) ,或35。对于这些 \( p,\frac{{3p} + {25}}{{2p} - 5} \) 的所有取值,均可为正整数。
问题20。解答:(D)。
若 \( \frac{{4n} + 5}{{7n} + 6} \) 为正整数,则存在某个公因数 \( m \) 能整除
\( {4n} + 5 \) 和 \( {7n} + 6 \) 两者。但此时 \( m \) 整除这些项的任意线性组合,因此 \( m \) 整除 \( j\left( {{4n} + 5}\right) + k\left( {{7n} + 6}\right) \) 。若令 \( j = 7 \) 和 \( k = - 4 \) (以消去 \( n \) ),我们得到 \( j\left( {{4n} + 5}\right) + k\left( {{7n} + 6}\right) = 7\left( {{4n} + 5}\right) - 4\left( {{7n} + 6}\right) = {35} - {24} = {11} \) 。由于
11为素数,因此我们有 \( m = {11} \) 。
于是 \( {4n} + 5 = {11t} \) ,其中 \( t \) 为任意正整数。
于是我们有
\( {4n} + 5 \equiv 0{\;\operatorname{mod}\;{11}} \)
\( {4n} \equiv 6{\;\operatorname{mod}\;{11}} \)
\( {4n} \equiv 6 + {11} + {11}{\;\operatorname{mod}\;{11}} \)
\( {4n} \equiv {28}{\;\operatorname{mod}\;{11}} \)
\( n \equiv 7{\;\operatorname{mod}\;{11}} \) .
从而我们得到 \( n = 7,{18},{29},{40} \) 。答案为(D)。
问题21。解答:(D)。
使用长除法,我们得到: \( \frac{{x}^{2} + {2x} + 5}{x - 3} = x + 5 + \frac{20}{x - 3} \)
\( x - 3 \) 必须是20的因数。20是20的最大因数,且由于我们要找最大整数值, \( x - 3 = {20} \) 和 \( x = {23} \) 。
问题22。解答:(D)。
若 \( \frac{n}{{40} - n} = {m}^{2} \) ,对某个整数 \( m \geq 0 \) ,则
\( n = \frac{{40}{m}^{2}}{{m}^{2} + 1} = \frac{{40}{m}^{2} + {40} - {40}}{{m}^{2} + 1} = {40} - \frac{40}{{m}^{2} + 1}. \)
由于 \( n \) 是整数, \( {m}^{2} + 1 \) 必须是 \( {40} = {2}^{3} \times 5 \) 的因数。
\( {m}^{2} + 1 = 1\; \Rightarrow \;m = 0 \Rightarrow \;n = 0 \) 和 \( \frac{n}{{40} - n} = 0. \)
\( {m}^{2} + 1 = 2\; \Rightarrow \;m = 1 \) 或 \( m = - 1\; \Rightarrow \;n = {20} \) 和 \( \frac{n}{{40} - n} = 1 \) 。
\( {m}^{2} + 1 = 4\; \) ( \( m \) 无整数解)。
\( {m}^{2} + 1 = 5\; \Rightarrow \;m = 2 \) 或 \( m = - 2\; \Rightarrow \;n = {32} \) 和 \( \frac{n}{{40} - n} = 4 \) 。
\( {m}^{2} + 1 = 8\; \) ( \( m \) 无整数解)。
\[ {m}^{2} + 1 = {10} \Rightarrow \;m = 3\text{ or }m = - 3\; \Rightarrow \;n = {36}\text{ and }\frac{n}{{40} - n} = 1. \]
\( {m}^{2} + 1 = {20}\; \) ( \( m \) 无整数解)。
\( {m}^{2} + 1 = {40}\; \) ( \( m \) 无整数解)。
\( n \) 的取值为0、20、32和36。
问题23。解答:(B)。
\[ {kx} = {72} + {3k} \Rightarrow \;x = \frac{72}{k} + 3. \]
仅当 \( k \) 是 \( {72.72} = {2}^{3} \times {3}^{2} \) 的因数时, \( x \) 才是整数。72的因数个数为 \( \left( {2 + 1}\right) \left( {3 + 1}\right) = {12} \) ,因此 \( k \) 有12个取值。
问题24。解答:(E)。
我们先考察项 \( {3n} + 1 \) 和 \( n \) 。
我们发现,对于任何整数 \( n \) , \( \frac{{3n} + 1}{n} = 3 + \frac{1}{n} \) 都不可约。
因此 \( {3n} + 1 \) 与 \( n \) 互质。
现在我们考察项 \( {3n} + 1 \) 和 \( {2n} - 1 \) 。
若 \( \frac{{3n} + 1}{{2n} - 1} \) 可约,则存在某个公因子 \( m \) 同时整除 \( {3n} + \) 1和 \( {2n} - 1 \) 。
根据定理2, \( m \) 也整除 \( j\left( {{3n} + 1}\right) + k\left( {{2n} - 1}\right) \) 。若令 \( j = 2 \) 和 \( k = - 3 \) (以便消去 \( n \) ),我们得到 \( 2\left( {{3n} + 1}\right) - 3\left( {{2n} - 1}\right) = 5 \) 。
由于5是素数,因此我们有 \( m = 5 \) 。
于是 \( {2n} - 1 = {5k} \) 对某个整数 \( k \) 成立。由于 \( n > 0 \) 且 \( {2n} - 1 \neq 0 \) , \( m \) 的最小值通过令 \( k = 1 \) 得到。因此 \( {2n} - 1 = 5 \Rightarrow n = 3 \) 。
问题25。解答:(D)。
\[ {2xy} - 2{x}^{2} + {3x} - {5y} + {11} = 0\; \Rightarrow \;y = \frac{2{x}^{2} - {3x} - {11}}{{2x} - 5} = x + 1 - \frac{6}{{2x} - 5} \]
由于 \( y \) 是整数, \( \frac{6}{{2x} - 5} \) 也必须是整数。
由于 \( {2x} - 5 \) 为奇数, \( {2x} - 5 \) 必须同时具有6的奇因子,因此我们有: \( {2x} - 5 = \pm 1;{2x} - 5 = \pm 3 \) 。
\( x = 3,2,4 \) 和1。 \( y \) 对应的值为-2、9、3和4。
(x, y)的解为 \( \left( {3, - 2}\right) ,\left( {2,9}\right) ,\left( {4,3}\right) \) ,以及(1,4)。
问题26。解答:(D)。
\[ \frac{{n}^{2} + 7}{n + 4} = \left( {n - 4}\right) + \frac{23}{n + 4} \]
当 \( n + 4 = {23p} \) ( \( p \) 为任意正整数)或 \( n = {23p} - 4,\frac{23}{n + 4} \) 可约时。我们知道 \( 1 \leq {23p} - 4 \leq {100} \) ,因此 \( p = 1,2,3,4 \) 且 \( n = {19},{42},{65} \) ,以及88。
问题27. 解答:54。
方法1(官方解答):
正整数 \( \mathrm{N} \) 是 \( {2004}^{2004} \) 的约数当且仅当 \( N = {2}^{i}{3}^{j}{167}^{k} \) 且 \( 0 \leq i \leq \) \( {4008},0 \leq j \leq {2004} \) ,且 \( 0 \leq k \leq {2004} \) 。这样的数恰有2004个正整数约数当且仅当 \( \left( {i + 1}\right) \left( {j + 1}\right) \left( {k + 1}\right) = {2004} \) 。因此满足条件的 \( N \) 的取值个数等于乘积为2004的正整数有序三元组的数量。无序三元组 \( \{ {1002},2,1\} ,\{ {668},3,1\} ,\{ {501},4,1\} ,\{ {334},6,1\} ,\{ {334},3,2\} ,\{ {167},{12},1\} ,\{ {167}, \) \( 6,2\} \) 和 \( \{ {167},4,3\} \) 各有6种排列方式,而三元组 \( \{ {2004},1 \) 、 \( 1\} \) 和 \( \{ {501},2,2\} \) 各有3种排列方式,因此总数为 \( 8 \times 6 + 2 \) \( \times 3 = {54} \) 。
方法2(我们的解答):
我们的解答基本思路与官方解答相同,但不采用枚举。
\( {2004} = 2 \times 2 \times 3 \times {167} \)
情况1:若所有因子均不为1,则因子为
2, 2, 3, 167
我们需要将其中两个数相乘得到第三个因子,剩余两个数即为另外两个因子。
例如,一种可能的组合是:
\( \left\{ {\left( {2 \times 2}\right) \;3\;{167}}\right\} \)
共有四种组合:
(2和2)、(2和3)、(2和167)以及(3和167)。对应的三元组为
\( \{ 4;{167};3\} ,\{ 6;{167};2\} ,\{ {334};3;2\} ,\{ {501};2;2\} \) .
有序三元组的数量: \( 6 + 6 + 6 + 3 = {21} \)
情况2:若三个因子中有一个为1,则数字为:1,2,2,3,167
现在我们知道其中一个因子必须是1,这意味着其余四个数需要组合成两个数。
换言之,我们需要计算将乘积 \( 2 \times 2 \times 3 \times {167} \) 拆分为两个因子的方式数量。
\( {2004} = 2 \times 2 \times 3 \times {167} \) 共有12个因子(含1和自身)。现在需要从总因子数中减去1和 \( 2 \times 2 \times 3 \times {167} \) ,得到10个可用因子。每个因子对应另一个因子,因此需要除以2。
设 \( d \) 为总因子数,则可用因子对的数量为
\[ \left\lfloor \frac{d - 2}{2}\right\rfloor = {10}/2 = 5\text{total pairs.} \]
以下是5个三元组的列表:
\( \{ {668};3;1\} ,\{ {501};4;1\} ,\{ {334};6;1\} ,\{ {167};{12};1\} ,\{ {1002};2;1\} \) ;
有序三元组的数量: \( 6 + 6 + 6 + 6 + 6 = {30} \)
情况3:如果有两个1作为因数,那么我们只有一种方式:\{1 1 2002\}
有序三元组的数量:3。
最终答案将是 \( {21} + {30} + 3 = {54} \) 。
在本章中,我们将讨论解决以下类型方程的常用方法:
文字方程、二次方程、绝对值方程、根式方程和有理方程。关于解指数方程的技巧,请参见第9章。
1. 解文字方程
文字方程是包含一个或多个字母的方程。这些字母是常数,但不是固定值。
一般情况: \( x \) 是方程中的变量。
情况I. 当 \( a \neq 0 \) 时,方程有唯一解: \( x = \frac{b}{a} \) 。
情况II. 当 \( a = b = 0 \) 时,方程有无穷多解。
情况III. 当 \( a = 0 \) 且 \( b \neq 0 \) 时,方程无解。
例1. 在公制中,温度用摄氏度( \( {}^{ \circ }\mathrm{C} \) )而非华氏度 \( \left( {{}^{ \circ }\mathrm{F}}\right) \) 来测量。公式如下: \( C = \frac{5\left( {F - {32}}\right) }{9} \) 。解 \( F \) 。
(A) \( \frac{9}{5}C + {32} \) (B) \( \frac{5}{9}C + {32} \) (C) \( \frac{9}{5}C - {32} \) (D) \( \frac{5}{9}C - {32} \) (E) 以上都不是
解:(A)。
两边同乘以 \( 9 : {9C} = 5\left( {F - {32}}\right) \)
去括号: \( {9C} = {5F} - 5 \times {32} \)
分离变量: \( {5F} = {9C} + 5 \times {32} \)
两边同除以 \( 5 : F = \frac{9}{5}C + {32} \) 。
例2. 若 \( x = \frac{a}{b + c} = \frac{b}{c + a} = \frac{c}{a + b} \) ,求 \( x \) 的值。
(A) \( \frac{1}{2} \) (B) -1 (C) \( \frac{1}{2} \) 或 -1 (D) 2 (E) \( - \frac{1}{2} \) 或 1
解:(C)。
情况I. 若 \( a + b + c = 0 \)
\( b + c = - a \)
\( x = \frac{a}{b + c} = \frac{a}{-a} = - 1 \)
情况II. 若 \( a + b + c \neq 0 \)
\( x = \frac{a}{b + c} = \frac{b}{c + a} = \frac{c}{a + b} = \frac{a + b + c}{b + c + c + a + a + b} = \frac{a + b + c}{2\left( {a + b + c}\right) } = \frac{1}{2}. \)
例3. 若 \( \frac{x}{y + z} = \frac{y}{x + z} = \frac{z}{y + x} \) ,求 \( \frac{x}{y + z} \) 的值。
(A) \( \frac{1}{2} \) (B) -1 (C) 2 (D) \( \frac{1}{2} \) 或 -1 (E) \( - \frac{1}{2} \) 或 1
解:(D)。
\[ \text{Let}\frac{x}{y + z} = \frac{y}{x + z} = \frac{z}{y + x} = k\text{.} \]
\[ \left. \begin{array}{l} x = k\left( {y + z}\right) \\ y = k\left( {x + z}\right) \\ z = k\left( {x + y}\right) \end{array}\right\} \]
将三个方程相加: \( x + y + z = {2k}\left( {x + y + z}\right) \) 。
我们需要讨论 \( x + y + z \) 的可能取值。
当 \( x + y + z \neq 0, k = \frac{1}{2} \Rightarrow \frac{x}{y + z} = \frac{1}{2} \) 。
当 \( x + y + z = 0,\; \Rightarrow \;x = - \left( {y + z}\right) \) 。
\( \frac{x}{y + z} = \frac{-\left( {y + z}\right) }{y + z} = - 1. \)
注意:每当你看到连比时,将每个比值设为 \( k \) 。
2. 求解二次方程
以下方程称为二次方程: \( a{x}^{2} + {bx} + c = 0 \) ,其中 \( a, b \) 、 \( c \) 为实数,且 \( a \neq 0 \) 。
平方根性质:方程 \( {x}^{2} = k \) 的解为 \( x = \sqrt{k} \) 和 \( x = - \sqrt{k} \) 。
二次公式: \( {x}_{1,2} = \frac{-b \pm \sqrt{{b}^{2} - {4ac}}}{2a} \) 。
判别式: \( \Delta = {b}^{2} - {4ac} \) 的值称为判别式(discriminant)。
\( \Delta = {b}^{2} - {4ac} \) | 二次方程有 |
\( \Delta = 0 \) | 一个实数解(两个相等解,重根) |
\( \Delta > 0 \) | 两个不相等的实数解 |
\( \Delta < 0 \) | 无实数解 |
例4. 解 \( x : x\left( {x - c}\right) = 1 - c \) 。
(A) \( x = 1,1 - c \) (B) \( x = 1, c \) (C) \( x = - 1, c + 1 \) (D) \( x = 1, c - 1 \) (E) 以上都不是
解答:(D)。
我们将给定方程改写为 \( {x}^{2} - {xc} + \left( {c - 1}\right) = 0 \) 。
利用二次公式, \( x = \frac{c \pm \sqrt{{c}^{2} - 4\left( {c - 1}\right) }}{2} = \frac{c \pm \left( {c - 2}\right) }{2} = c - 1,1 \) 。
例5. 求方程 \( {x}^{4} - {34}{x}^{2} + {225} = 0 \) 的最大解。
(A) -5 (B) 5 (C) \( 5\sqrt{2} \) (D) 8 (E) 10
解答:(B)。
设 \( {x}^{2} = y \) 。原方程可写为 \( {y}^{2} - {34y} + {225} = 0 \)
\( \left( {y - 9}\right) \left( {y - {25}}\right) = 0 \) .
\( y = 9 \Rightarrow \;{x}^{2} = 9\; \Rightarrow \;x = 3 \) 或 \( x = - 3. \)
或 \( y = {25}\; \Rightarrow \;{x}^{2} = {25}\; \Rightarrow \;x = 5 \) 或 \( x = - 5 \) 。
最大解为5。
例6. (2002 AMC 10 B) 设 \( a \) 和 \( b \) 为非零实数,且方程 \( {x}^{2} + {ax} + b = 0 \) 的解为 \( a \) 和 \( b \) 。则数对(a, b)为? (B)(-1,2) (C)(1,-2) (D)(2,-1)
解答:(C)。
方法1(官方解答):
给定条件意味着 \( {x}^{2} + {ax} + b = \left( {x - a}\right) \left( {x - b}\right) = {x}^{2} - \left( {a + b}\right) x + {ab} \) ,因此 \( a + b = - a \) 且 \( {ab} = b \) 。
由于 \( b \neq 0 \) ,第二个方程意味着 \( a = 1 \) 。第一个方程给出 \( b = - 2, \) ,因此 \( \left( {a, b}\right) = \left( {1, - 2}\right) . \)
方法二(我们的解法):
假设 \( a \) 和 \( b \) 是解,我们有:
\[ {a}^{2} + a \times a + b = 0 \tag{1} \]
\[ {b}^{2} + a \times b + b = 0 \tag{2} \]
(1)-(2): \( {a}^{2} - {b}^{2} + a \times a - a \times b = 0\; \Rightarrow \;\left( {a - b}\right) \left( {a + b}\right) + a\left( {a - b}\right) = 0 \)
\( \Rightarrow \;\left( {a - b}\right) \left\lbrack {\left( {a + b}\right) + a}\right\rbrack = 0 \) (3)
由于 \( b \neq 0,\left( 2\right) \) 也可以写成 \( b + a = - 1 \) (4)
将(4)代入(3): \( \left( {a - b}\right) \left( {a - 1}\right) = 0 \)
所以我们有 \( a = 1 \) 或 \( a = b \) 。
将 \( a = 1 \) 代入(1),我们得到 \( {1}^{2} + 1 \times 1 + b = 0 \Rightarrow b = - 2 \) ,因此 \( \left( {a, b}\right) = \left( {1, - 2}\right) \) 。
答案是(C),我们可以忽略 \( a = b \) 的情况,因为在五个选项中没有这个选择。
例7.(2005 AMC 10 A)有两个 \( a \) 的值使得方程 \( 4{x}^{2} + {ax} + {8x} + 9 = 0 \) 对于 \( x \) 只有一个解。这两个 \( a \) 值的和是多少?
(A) -16 (B) -8 (C) 0 (D) 8 (E) 20
解答:(A)。
方法一(官方解法):
二次公式给出 \( {x}_{1,2} = \frac{-\left( {a + 8}\right) \pm \sqrt{{\left( a + 8\right) }^{2} - 4 \cdot 4 \cdot 9}}{2 \cdot 4} \) 。
当判别式的值 \( {\left( a + 8\right) }^{2} - {144} \) 为0时,方程恰好有一个解。这意味着 \( a = - {20} \) 或 \( a = 4 \) ,其和为-16。
方法二(官方解法):
当且仅当该多项式是一个二项式的平方,且一次项为 \( \pm \sqrt{4{x}^{2}} = \pm {2x} \) 、常数项为 \( \pm \sqrt{9} = \pm 3 \) 时,方程有一个解。因为 \( \left( {\left( 2x \pm 3\right) }^{2}\right) \) 的一次项为 \( \pm {12x} \) ,所以 \( a + 8 = \pm {12} \) 。于是 \( a \) 为-20或4,这些值的和为-16。方法三(我们的解法):方程只有一个解意味着 \( \Delta = {b}^{2} - {4ac} = 0\; \Rightarrow \;{\left( a + 8\right) }^{2} - 4 \times 4 \times 9 = 0 \Rightarrow \;{\left( a + 8\right) }^{2} = {\left( {12}\right) }^{2}. \) \( a = {12} - 8 = 4 \) 或 \( a = - {12} - 8 = - {20}. \) ,所以答案是 \( 4 - {20} = - {16} \) 。
例8. 求最小的整数值 \( k \) ,使得 \( {2x}\left( {{kx} - 6}\right) - {x}^{2} + 8 = \) 0无实根?
(A) -1 (B) 2 (C) 3 (D) 4 (E) 5
解答:(C)。
将二次式写成标准形式: \( \left( {{2k} - 1}\right) {x}^{2} - {12x} + 8 = 0 \) ,可知判别式 \( D \) 为 \( {144} - 4\left( {{2k} - 1}\right) 8 = {176} - {64k} = {16}\left( {{11} - {4k}}\right) \) 。当且仅当判别式为负时,二次方程无实根。 \( D \) 为负